I am reading a PHD thesis online "A controlled quantum system of individual neutral atom" by Stefan Kuhr. In it on pg46, he has a Hamiltonian
I am also reading a book by L. Allen "optical resonance and two level atoms" in it on page 34 he starts with a Hamiltonian where the perturbation is...
Well, I have no clues for this problem.
Since I can get nothing from the definition of ##\rho##, I tried from the right part.
Also, I know that ##\left ( \vec r \cdot \vec \sigma \right ) ^2={r_1}^2 {\sigma _1}^2+{r_2}^2 {\sigma _2}^2+{r_3}^2 {\sigma _3}^2##.
Plus, ##\rho## is positive; then...
I've read that ##\left | \psi \right > =cos \frac \theta 2 \left | 0 \right > + e^{i \phi} sin \frac \theta 2 \left | 1 \right >##, and the corresponding point in the Bloch sphere is as the fig below shows.
I think ##\left | 0 \right >## and ##\left | 1 \right >## are orthonormal vectors...
Anyone know how to change a basis of a qubit state of bloch sphere given a general qubit state? There are 3 different basis corresponding to each direction x,y,z where |1> ,|0> is the z basis, |+>, |-> is the x basis and another 2 ket notation for y basis.
Given a single state in the x basis...
I am pretty sure that I would be comparing apples and oranges in this question, but as I usually learn something from the responses telling me in detail that my question is silly, here goes: Does the phase used as a weight in Feynman's path integral formulation (i.e., the quantum action S in...
If I construct a set of qubit gates, say {G1, G2 ... Gk ... Gn}, that can act on a state |ψ>, what does it mean for the set of states Gk |ψ> to span the Bloch sphere?
As an example, take the set {G1, G2, G3, G4} = { I, X π/2 , Y π/2, Xπ }
Here, X π/2 denotes a π/2 rotation about the x-axis, Y...
Hey,
(I have already asked the question at http://physics.stackexchange.com/questions/244586/bloch-sphere-interpretation-of-rotations, I am not sure this forum's etiquette allows that!)
I am trying to understand the following statement. "Suppose a single qubit has a state represented by the...
I found a funny model of the qubit written by Aerts in
Foundations of quantum physics: a general
realistic and operational realistic and operational approach.
At the beginning the qubit is at the point P on the Bloch sphere. It will be measured along another direction (two opposite points on...
This question is mostly about group theory but I would like to understand it in the context of qubits rotating in a Bloch Sphere.
What my understanding of things are right now:
In the rotation Lie Group ##SO(3)##, we have three free parameters (##\frac{n(n-1)}{2}##), and this is also why we end...
Can anyone explain to me why the following operators are rotation operators:
\begin{align*}R_x(\theta) &= e^{-i\theta X/2}=\cos(\frac{\theta}{2})I-i\sin(\frac{\theta}{2})X=
\left(\!\begin{array}{cc}\cos(\frac{\theta}{2}) & -i\sin(\frac{\theta}{2}) \\ -i\sin(\frac{\theta}{2})&...
Homework Statement
What is reduced density matrix ##\rho_A## and the Bloch vector representation for a state that is 50% ##|0 \rangle## and 50% ##\frac{1}{\sqrt{2}}(|0 \rangle + |1 \rangle)##Homework Equations
The Attempt at a Solution
[/B]
I haven't seen many (any?) examples of this so I'm...
Homework Statement
The problem is as follows. I have two spins, m_S and m_I. The first spin can either be \uparrow or \downarrow , and the second spin can be -1, 0 or 1.
Now, I envision the situation as the first spin being on the bloch sphere, with up up to and down at the bottom.
What I...
I'm having a bit of a brain fart here, so hopefully someone can help.
Consider a closed, two-level quantum system. We know we can describe pure states as
\alpha |0\rangle + \beta |1 \rangle
for some orthonormal basis |0\rangle, |1 \rangle . The normalization conditions means we can...
I have a general question about extracting information from measurement of a qubit. Theoretically a qubit in a superposition state contains an infinite amount of information, but when measured collapses to a definite state and result. My question is this:
Is there a way to obtain a value from...
Hey guys,
I'm attempting to map some discrete points on the surface of the Bloch sphere:
For instance, the full spectrum of ranges for variable theta is 0 < theta < pi. However, my goal is to limit that range from some theta_1 < theta < theta_2. I was going to use a spherical harmonic...
There is something that I don't quite understand in relation to the Bloch Sphere representation of qubits. I've read that any vector on the sphere is a superposition of two basic states, like spin up and spin down, denoted by |1> and |0>.
So does this mean that if the vector is at z=0...
So I tried learning about spinors yesterday, and got myself confused. Hopefully someone can tell me if I'm barking up the right tree...
The way they were introduced was by exhibiting a homomorphism from C^3 to C^2 by using the dot product:
(x1, y1, z1) . (x2, y2, z2) = x1*x2 + y1*y2 +...
The Bloch sphere helps understanding the mathematical results for a one-spin state. One could think of the state as a spin pointing in direction \hat{n}. Then the probability for measureing the spin in the direction \hat{m} is simply
P=|<\hat{m}|\hat{n}>|^2=\frac{1+\hat{n}\cdot\hat{m}}{2}
and...
I am reading Quantum Computation and Quantum Information by Nelson and Chuang myself and came across the Bloch Sphere representation of a quibit on page 15 (equation 1.4) as:
|\psi> = \cos\frac{\theta}{2} |0> + e^{i\psi}\sin\frac{\theta}{2} |1>
I have two questions:
1. What is the motivation...
I have one critical problem with quantum computing.
When I have one quantum state on bloch sphere, I do some transformation on that state for example PauliX transformation.
As far as I know, we can present quantum state as
|y> = cos (theta/2) |0> + e^(i*phi) sin (theta/2) |0>
So if we...